Is Vieta's practical?

Algebra Level 5

25 ( 4 a 1 2 + 1 ) ( 4 a 2 2 + 1 ) ( 4 a 5 2 + 1 ) 25(4a_1^2 + 1)(4a_2^2 + 1)\cdots (4a_{5}^2 + 1)

Let a 1 , a 2 , , a 5 a_1, a_2, \ldots,a_{5} denote the roots to the equation m = 1 5 m x m = 0 \displaystyle \sum_{m=1}^{5} m x^m = 0 .

Find the value of the expression above.


The answer is 145.

This section requires Javascript.
You are seeing this because something didn't load right. We suggest you, (a) try refreshing the page, (b) enabling javascript if it is disabled on your browser and, finally, (c) loading the non-javascript version of this page . We're sorry about the hassle.

5 solutions

Aareyan Manzoor
Dec 17, 2015

neclect the x=0 solution. 4 0 2 + 1 = 1 4*0^2+1=1 which doesnt affect the product. f ( x ) = 5 x 4 + 4 x 3 + 3 x 2 + 2 x + 1 = 5 ( x a 1 ) ( x a 2 ) ( x a 3 ) ( x a 4 ) f(x)=5x^4+4x^3+3x^2+2x+1=5(x-a_1)(x-a_2)(x-a_3)(x-a_4) . remember this:the basic definition of a polynomial.we have the given expression equal to 25 ( 4 4 ( i 2 a 1 ) ( i 2 a 1 ) ( i 2 a 2 ) ( i 2 a 2 ) . . . ( i 2 a 4 ) ( i 2 a 4 ) ) = 4 4 ( 5 ( i 2 a 1 ) ( i 2 a 2 ) . . . . ( i 2 a 4 ) ) ( 5 ( i 2 a 1 ) ( i 2 a 2 ) . . . . ( i 2 a 4 ) ) 25(4^4(\frac{i}{2}-a_1)(-\frac{i}{2}-a_1)(\frac{i}{2}-a_2)(-\frac{i}{2}-a_2)...(\frac{i}{2}-a_{4})(-\frac{i}{2}-a_{4}))=4^4(5(\frac{i}{2}-a_1)(\frac{i}{2}-a_2)....(\frac{i}{2}-a_{4}))(5(-\frac{i}{2}-a_1)(-\frac{i}{2}-a_2)....(-\frac{i}{2}-a_{4})) this is equal to by our above def: 4 4 f ( i 2 ) f ( i 2 ) 4^4f(\frac{i}{2})f(-\frac{i}{2}) it is not hard to compute, you can use AGP or differentiate GP. either ways, the answer will just be 4 4 ( 8 + 9 i 16 ) ( 8 9 i 16 ) = 8 2 + 9 2 = 145 4^4(\frac{-8+9i}{16})(\frac{-8-9i}{16})=8^2+9^2=\boxed{145}

THANKYOU AGAIN!!

Pi Han Goh - 5 years, 5 months ago

Log in to reply

My solution was not very elegant but I did get estimates for the individual factors:

4 a 1 2 + 1 = 1.643582558 + 1.541576543 i 4*a_1^2+1 = 1.643582558 + 1.541576543*i

4 a 2 2 + 1 = 1.643582558 1.541576543 i 4*a_2^2+1 = 1.643582558 - 1.541576543*i

4 a 3 2 + 1 = . 7635825578 + . 7477724975 i 4*a_3^2+1 = -.7635825578 + .7477724975*i

4 a 4 2 + 1 = . 7635825578 . 7477724975 i 4*a_4^2+1 = -.7635825578 - .7477724975*i

4 a 5 2 + 1 = 1 4*a_5^2+1 = 1

The expression we had to evaluate was found on my TI84+ calculator as 145 + 0 i 145+0*i

If you want to look at the individual roots:

a 1 = . 5378322749 + . 3582846863 i a_1 = .5378322749 + .3582846863*i

a 2 = . 5378322749 . 3582846863 i a_2 = .5378322749 - .3582846863*i

a 3 = . 1378322749 + . 6781543891 i a_3 = .1378322749 + .6781543891*i

a 4 = . 1378322749 . 6781543891 i a_4 = .1378322749 - .6781543891*i

a 5 = 0 a_5 = 0

Bob Kadylo - 5 years, 5 months ago

Log in to reply

Good first try. The hard part is to prove that the answer is exactly equals to 145.

Pi Han Goh - 5 years, 5 months ago

Log in to reply

@Pi Han Goh I agree !! My old brain is having trouble grasping some of the elegant proofs and solutions I see on Brilliant. I am also trying to learn how to use LaTeX. I wish I was 40 years younger!

Bob Kadylo - 5 years, 5 months ago

Log in to reply

Log in to reply

@Pi Han Goh WOW !!! Thank you very much for this super tip !!

Bob Kadylo - 5 years, 5 months ago

did you multiply by 25?

Joel Yip - 5 years, 3 months ago

Log in to reply

@Joel Yip What I showed were the factors and raw roots before 25 was utilized.

Bob Kadylo - 5 years, 3 months ago

Log in to reply

@Bob Kadylo i meant did you multiply the 4(raw roots)^2+1 together with the 25?

Joel Yip - 5 years, 3 months ago

Log in to reply

@Joel Yip Yes, I did - to get the value of the expression so I could submit the answer of 145. Just did it again to make sure. Still works.

Bob Kadylo - 5 years, 3 months ago

I did the same method 2

Righved K - 5 years, 5 months ago

I think the last line, to the left of the answer box, should have both denominators of 32 changed to 16. Then the 4 4 4^4 in front would cancel the product of 16 16 16*16 across the denominators. This comment refers to the solution above by Aareyan Monzoor.

Bob Kadylo - 5 years, 5 months ago

@Pi Han Goh

How is one going to use Vieta's?

Joel Yip - 5 years, 3 months ago

Log in to reply

Hint : If a polynomial f ( x ) f(x) has roots a 1 , a 2 , , a n a_1, a_2, \ldots, a_n , then the polynomial for f ( x ) f(\sqrt x ) has roots a 1 2 , a 2 2 , a n 2 a_1 ^2 ,a_2 ^2 \ldots ,a_n ^2 .

Pi Han Goh - 5 years, 3 months ago

Log in to reply

(I think. Might be wrong)

That's not exactly Vieta's. That's just "transformation of roots".

Yes, sometimes transformation of roots is done via evaluating coefficient-wise using Vieta's. But in your case, you are proposing a direct algebraic calculation (IE shifting and squaring) that avoids vieta.

Calvin Lin Staff - 5 years, 2 months ago

Log in to reply

@Calvin Lin Yeah. To be more precise, what I've done is transformation of roots, then Vieta's formula.

Pi Han Goh - 5 years, 2 months ago
Jake Lai
Dec 18, 2015

(Adapted from Aareyan Manzoor's method. Rewritten for clarity.)

Notice that the given equation has a root x = 0 x=0 , which can be ignored since in the given expression, 4 ( 0 ) 2 + 1 = 1 4(0)^2+1 = 1 , which is the multiplicative identity. Hence let a 5 = 0 a_5 = 0 , and f ( x ) = m = 1 5 m x m 1 = 5 n = 1 4 ( x a n ) \displaystyle f(x) = \sum_{m=1}^5 mx^{m-1} = 5 \prod_{n=1}^4 (x-a_n) .

Applying the identity ( a + i b ) ( a i b ) = a 2 + b 2 (a+ib)(a-ib) = a^2+b^2 , we can reform the given expression:

25 n = 1 4 ( 4 a n 2 + 1 ) = 25 n = 1 4 ( 2 a n + i ) ( 2 a n i ) = 25 n = 1 4 [ 2 ( i 2 a n ) ] n = 1 4 [ 2 ( i 2 a n ) ] = [ 2 4 f ( i 2 ) ] [ 2 4 f ( i 2 ) ] = 2 8 f ( i 2 ) f ( i 2 ) . \begin{aligned} 25 \prod_{n=1}^4 (4a_n^2+1) &= 25 \prod_{n=1}^4 (2a_n+i)(2a_n-i) \\ &= 25 \prod_{n=1}^4 \left[ 2 \left( -\frac{i}{2}-a_n \right) \right] \prod_{n=1}^4 \left[ 2 \left( \frac{i}{2}-a_n \right) \right] \\ &= \left[ 2^4 f \left( -\frac{i}{2} \right) \right] \left[ 2^4 f \left( \frac{i}{2} \right) \right] \\ &= 2^8 f \left( -\frac{i}{2} \right) f \left( \frac{i}{2} \right). \end{aligned}

As has been noted, the above may be computed easily. For variety's sake, I will present a method besides AGP/differentiating GP, which uses the aforementioned identity:

2 8 f ( i 2 ) f ( i 2 ) = 2 8 [ m = 1 5 m ( i 2 ) m 1 ] [ m = 1 5 m ( i 2 ) m 1 ] = 2 8 [ 5 ( 1 2 ) 4 3 ( 1 2 ) 2 + 1 + 4 i ( 1 2 ) 3 2 i ( 1 2 ) ] [ 5 ( 1 2 ) 4 3 ( 1 2 ) 2 + 1 4 i ( 1 2 ) 3 + 2 i ( 1 2 ) ] = 1 6 2 [ ( 5 16 3 4 + 1 ) 2 + ( 4 8 2 2 ) 2 ] = ( 5 12 + 16 ) 2 + ( 8 16 ) 2 = 9 2 + 8 2 = 145 . \begin{aligned} 2^8 f \left( -\frac{i}{2} \right) f \left( \frac{i}{2} \right) &= 2^8 \left[ \sum_{m=1}^5 m\left(-\frac{i}{2}\right)^{m-1} \right] \left[ \sum_{m=1}^5 m\left(\frac{i}{2}\right)^{m-1} \right] \\ &= 2^8 \left[ 5\left(\frac{1}{2}\right)^4 - 3\left(\frac{1}{2}\right)^2 + 1 + 4i\left(\frac{1}{2}\right)^3 - 2i\left(\frac{1}{2}\right)\right] \left[ 5\left(\frac{1}{2}\right)^4 - 3\left(\frac{1}{2}\right)^2 + 1 - 4i\left(\frac{1}{2}\right)^3 + 2i\left(\frac{1}{2}\right)\right] \\ &= 16^2 \left[ \left( \frac{5}{16} - \frac{3}{4} + 1 \right)^2 + \left( \frac{4}{8} - \frac{2}{2} \right)^2 \right] \\ &= \left( 5 - 12 + 16 \right)^2 + \left( 8 - 16 \right)^2 \\ &= 9^2+8^2 \\ &= \boxed{145}. \end{aligned}

Shaurya Gupta
Dec 19, 2015

Here's another solution without using complex numbers:
One of the roots of the polynomial is 0 0 which leaves us with 5 x 4 + 4 x 3 + 3 x 2 + 2 x + 1 = 5 ( x a 1 ) ( x a 2 ) ( x a 3 ) ( x a 4 ) 5x^4+4x^3+3x^2+2x+1 = 5(x-a_1)(x-a_2)(x-a_3)(x-a_4)

d d x ( 1 + x + . . . + x 5 ) = 5 x 6 6 x 5 + 1 ( x 1 ) 2 = 5 ( x a 1 ) ( x a 2 ) ( x a 3 ) ( x a 4 ) \implies \frac{d}{dx}(1+x+...+x^5)=\frac{5x^6-6x^5+1}{(x-1)^2}=5(x-a_1)(x-a_2)(x-a_3)(x-a_4)

5 x 6 6 x 5 + 1 = 5 ( x 1 ) 2 ( x a 1 ) ( x a 2 ) ( x a 3 ) ( x a 4 ) 5x^6-6x^5+1 = 5(x-1)^2(x-a_1)(x-a_2)(x-a_3)(x-a_4)
Replacing x x in 5 x 6 6 x 5 + 1 5x^6-6x^5+1 with x \sqrt{x} and rationalising, we get 25 x 6 36 x 5 + 10 x 3 + 1 = 25 ( x 1 ) 2 ( x a 1 2 ) ( x a 2 2 ) ( x a 3 2 ) ( x a 4 2 ) 25x^6-36x^5+10x^3+1 = 25(x-1)^2(x-a_1^2)(x-a_2^2)(x-a_3^2)(x-a_4^2)
Substitute x = 1 4 x= {-1\over 4} to get the required value.

SWEEEEET! Upvoted

Pi Han Goh - 5 years, 5 months ago
Chew-Seong Cheong
Dec 21, 2015

m = 1 5 m x m = 0 x + 2 x 2 + 3 x 3 + 4 x 4 + 5 x 5 = 0 x ( 5 x 4 + 4 x 3 + 3 x 2 + 2 x + 1 ) = 0 \begin{aligned} \sum_{m=1}^5 mx^m & = 0 \\ \Rightarrow x + 2x^2 + 3x^3 + 4x^4 + 5x^5 & = 0 \\ x(5x^4 + 4x^3 + 3x^2 + 2x + 1) & = 0 \end{aligned}

This implies that one of the roots is 0 0 and let it be a 5 = 0 a_5 = 0 . Then we have:

\begin{aligned} 25 \prod_{m=1}^5 (4a_m^2 +1) & = 25 \prod_{m=1}^\color{#3D99F6}{4} (4a_m^2 +1) \quad \quad \quad \quad \quad \quad \small \color{#3D99F6}{\text{Since }4a_5^2 + 1 = 1} \\ & = 25 \prod_{m=1}^4 (2a_m^2 +i) (2a_m^2 -i) \\ & = 25 \prod_{m=1}^4 (2a_m^2 +i) \prod_{m=1}^4 (2a_m^2 -i) \\ & = 25 \left(16\prod_{m=1}^4 a_m +8i\sum_{cyc}a_1a_2a_3 -4\sum_{cyc}a_1a_2 -2i\sum_{m=1} ^4 a_m + 1 \right) \\ & \quad \quad \quad \times \left(16\prod_{m=1}^4 a_m - 8i\sum_{cyc}a_1a_2a_3 -4\sum_{cyc}a_1a_2 + 2i\sum_{m=1} ^4 a_m + 1 \right) \\ & = 25 \left(16 \color{#3D99F6}{\left(\frac{1}{5}\right)} +8i\color{#3D99F6}{\left(-\frac{2}{5}\right)} -4 \color{#3D99F6}{\left(\frac{3}{5}\right)} -2i\color{#3D99F6}{\left(-\frac{4}{5}\right)} + 1 \right) \\ & \quad \quad \quad \times \left(16 \color{#3D99F6}{\left(\frac{1}{5}\right)} - 8i\color{#3D99F6}{\left(-\frac{2}{5}\right)} -4 \color{#3D99F6}{\left(\frac{3}{5}\right)} + 2i\color{#3D99F6}{\left(-\frac{4}{5}\right)} + 1 \right) \quad \quad \small \color{#3D99F6}{\text{By Vieta's formulas}} \\ & = \frac{25}{25}(16 - 16i - 12 + 8i + 5) (16 + 16i - 12 - 8i + 5) \\ & = (9 - 8i)(9+8i) = 81 + 64 = \boxed{145} \end{aligned}

quite a nice solution ehh.

Joel Yip - 5 years, 3 months ago
Rahul Saxena
Dec 21, 2015

GREAT WORK!

Pi Han Goh - 5 years, 5 months ago

Log in to reply

Again I made an arithmetic error (like with Vieta's Take Too Long). Multiply 1 + 2 x + 3 x 2 + 4 x 3 + 5 x 4 = 0 1+2x+3x^2+4x^3+5x^4=0 and 1 2 x + 3 x 2 4 x 3 + 5 x 4 = 0 1-2x+3x^2-4x^3+5x^4=0 to get 25 x 8 + 14 x 6 + 3 x 4 + 2 x 2 + 1 = 0 25x^8+14x^6+3x^4+2x^2+1=0 . Then the answer is 25 4 ( 14 ) + 4 2 ( 3 ) 4 3 ( 2 ) + 4 4 = 145 25-4(14)+4^2(3)-4^3(2)+4^4=145 .

James Wilson - 3 years, 7 months ago

Log in to reply

How do you know you need to multiply these 2 quintic equations?

Pi Han Goh - 3 years, 7 months ago

Log in to reply

@Pi Han Goh (Take a 5 = 0 a_5=0 for obvious reasons.) In terms of the roots, the product of those equations will take the form 25 ( x 2 a 1 2 ) ( x 2 a 2 2 ) ( x 2 a 3 2 ) ( x 2 a 4 2 ) 25(x^2-{a_1}^2)(x^2-{a_2}^2)(x^2-{a_3}^2)(x^2-{a_4}^2) , since all the roots of one will be the negative of the roots of the other. Then apply Vieta's formulas. Whenever the sum of the product of the squares of roots has an odd number of the factors in each product (i.e., has a negative sign in the front in Vieta's formulas), then negating the corresponding coefficient will give the sum over product of n roots (or squares of roots) as it appears in the expression of interest after it is expanded (not including the power of 4 in front). For the ones with the positive sign in front (i.e. with an even number of factors in the product it is being summed over in Vieta's formulas), simply take the coefficients corresponding to those as is. These numbers can then be substituted straight in. There's not really any complicated algebraic manipulation involved.

James Wilson - 3 years, 7 months ago

Log in to reply

@James Wilson Nicely done!

Pi Han Goh - 3 years, 7 months ago

0 pending reports

×

Problem Loading...

Note Loading...

Set Loading...